What is Limit: Definition and 1000 Discussions

In mathematics, the limit inferior and limit superior of a sequence can be thought of as limiting (i.e., eventual and extreme) bounds on the sequence. They can be thought of in a similar fashion for a function (see limit of a function). For a set, they are the infimum and supremum of the set's limit points, respectively. In general, when there are multiple objects around which a sequence, function, or set accumulates, the inferior and superior limits extract the smallest and largest of them; the type of object and the measure of size is context-dependent, but the notion of extreme limits is invariant.
Limit inferior is also called infimum limit, limit infimum, liminf, inferior limit, lower limit, or inner limit; limit superior is also known as supremum limit, limit supremum, limsup, superior limit, upper limit, or outer limit.

The limit inferior of a sequence




x

n




{\displaystyle x_{n}}
is denoted by





lim inf

n





x

n




or





lim
_



n






x

n


.


{\displaystyle \liminf _{n\to \infty }x_{n}\quad {\text{or}}\quad \varliminf _{n\to \infty }x_{n}.}
The limit superior of a sequence




x

n




{\displaystyle x_{n}}
is denoted by





lim sup

n





x

n




or





lim
¯



n






x

n


.


{\displaystyle \limsup _{n\to \infty }x_{n}\quad {\text{or}}\quad \varlimsup _{n\to \infty }x_{n}.}

View More On Wikipedia.org
  1. terryds

    How Do You Solve This Trigonometric Limit Problem?

    Homework Statement [/B] ##\lim x\rightarrow \frac{\pi }{4} (\frac{1-\tan x}{\sin x - \cos x})## The Attempt at a Solution [/B] By assuming y = x-π/4 , the limit become : ## \lim y\rightarrow 0 (\frac{1- \tan (y+\frac{\pi}{4})}{\sin (y+\frac{\pi}{4}) - \cos (y+\frac{\pi}{4})}) = \lim...
  2. terryds

    What is the Limit at Infinity for (2^x-5^x) / (3^x+5^x)?

    Homework Statement lim x->∞ (2^x-5^x) / (3^x+5^x) Choices : a. -1 b. -2/3 c. 1 d. 6 e. 25 2. The attempt at a solution Hmmm.. I really have no idea about this.. This is an unusual problem.. Please tell me...
  3. Dethrone

    MHB Limit of (n)^(1/n)/n as n approaches infinity

    Determine $$\lim_{{n}\to{\infty}}\frac{(n!)^{1/n}}{n}$$
  4. Dethrone

    MHB Limit 2^n/n using epsilon delta

    Using epsilon delta, prove $$\lim_{{n}\to{\infty}}\frac{2^n}{n!}=0$$ Doesn't seem too difficult, but I have forgotten how to do it. Obvious starting point is $\forall \epsilon >0$, $\exists N$ such that whenever $n>N,\left|\frac{2^n}{n!} \right|<\epsilon$.
  5. Alex_Neof

    Get a Clue: Understanding the Limit of a Series

    Homework Statement I'm reading a derivation and there is a step where the writer goes from: ## \sum_{n=0}^\infty e^{-n\beta E_0}## to: ## \frac {1} {(1-e^{-\beta E_0})}.## I can't see how they did this.Homework Equations [/B] I think it just involves equation manipulation. The Attempt at...
  6. terryds

    What Is the Limit of This Trigonometric Expression as x Approaches π/2?

    Homework Statement Find the limit of : lim x-> (π/2) (2-2sin x)/(6x-3π) 2. The attempt at a solution lim x-> (π/2) (2-2sin x)/(6x-3π) =lim x-> (π/2) 2-2 sin x / 6 (x- (1/2)pi) Assuming that y = x - (π/2) So, lim y->0 (2-2sin(y+pi/2))/6y lim y->0 (2-2 (sin y cos pi/2 + cos y sin pi/2)/6y...
  7. Y

    Understanding Limit Definition and the Role of Inequalities in Calculus

    Homework Statement It is not exactly a homework question, but why does the definition of a limit use strict inequalities as follows: if 0 < |x - a| < δ, then |f(x) - l| < ε rather than weak inequalities, for example if 0 < |x - a| < δ, then |f(x) - l| ≤ ε Could the addition of the equality...
  8. vktsn0303

    Prove Concept of Limit: n2-1/(n2 + n + 1)→1

    How can it be proved that as lim n tends to infinity, (n2-1)/(n2 + n + 1) tends to 1 ?
  9. StrangeCharm

    Finding the Limit of a Convergent Sequence

    Homework Statement Determine whether the sequence converges or diverges. If it converges, find the limit. Here's the sequence: http://www4a.wolframalpha.com/Calculate/MSP/MSP89541ea2ag9dg617bcd6000050d52e94i67ei593?MSPStoreType=image/gif&s=39&w=66.&h=44. Homework Equations N/A The Attempt at...
  10. L

    Central limit theorem and estimates of probability

    Homework Statement Assume five hundred people are given one question to answer - the question can be answered with a yes or no. Let p =the fraction of the population that answers yes. Give an estimate for the probability that the percent of yes answers in the five hundred person sample is...
  11. S

    Why Are Assumptions Critical in the Limit of Composite Functions?

    I need help with the following theorem: Let I, J ⊆ℝ be open intervals, let x∈I, let g: I\{x}→ℝ and f: J→ℝ be functions with g[I\{x}]⊆J and Limz→xg(x)=L∈J. Assume that limy→L f(y) exists and that g[I\{x}]⊆J\{g(x)},or, in case g(x)∈g[I\{x}] that limy→L f(y)=f(L). Then f(g(x)) converges at x, and...
  12. terryds

    Limit of tan(x)/x as x approaching zero

    The hint I found in http://math.stackexchange.com/questions/448207/how-to-prove-that-lim-limits-x-to0-frac-tan-xx-1#answer-448210 limx→0(tan(x) / x)= limx→0( (tan(x)−0) / (x−0)) = limx→0 ( (tan(x)−tan(0) ) / (x−0) )=⋯ Then, I don't know how to continue it.. What identity is used ? I don't see...
  13. nuuskur

    Proof: limit of product is the product of limits

    Homework Statement Let f_1,f_2\colon\mathbb{R}^m\to\mathbb{R} and a cluster point P_0\in D\subset\mathbb{R}^m (domain) Prove that \lim_{P\to P_0} f_1(P)\cdot f_2(P) = \lim_{P\to P_0} f_1(P)\cdot\lim_{P\to P_0} f_2(P) Homework EquationsThe Attempt at a Solution Let \begin{cases} \lim_{P\to...
  14. C

    The Unique Limit of a Complex Function

    Homework Statement I'm struggling with the proof that the limit of a complex function is unique. I'm struggling to see how |L-f(z*)| + |f(z*) - l'| < ε + ε is obtained. Homework Equations 0 < |z-z0| < δ implies |f(z) - L| < ε, where L is the limit of f(z) as z→z0 .The Attempt at a Solution...
  15. U

    Weak Gravity & Newtonian Limit: Letting g^kmu = eta^kmu

    Assume we have a free-falling particle in gravity in a static metric. Its worldline is described by: g_{\mu \nu} = \eta_{\mu \nu} + h_{\mu \nu} where ##|h_{\mu \nu} << 1|##. Taken from Hobson's book: Why did they let ##g^{k\mu} = \eta^{k\mu}##?
  16. D

    MHB Fundamental theorem and limit proofs

    Prove that the limit as n approaches infinity of ((2^n * n!)/n^n) equals to zero. The hint is to use Stirling's approximation. What is this?
  17. slatts

    Do M or string theory imply a lower limit to size?

    In his 2001 Three Roads to Quantum Gravity, on its p.l66, Smolin says, "M theory, if it exists, cannot describe a world in which space is continuous and one can pack an infinite amount of information into any volume, no matter how small." As a lay person, I'm hoping to get an informed opinion...
  18. M

    MHB Does the Limit Exist? Check to Find Out!

    Hey! :o I have found applying De L'Hoptal's Rule that $$\lim_{x \rightarrow 0} \frac{\sin 2x-2x}{x^3}=-\frac{4}{3}$$ Now I am asked whether the limit $$\lim_{(x, y) \rightarrow (0, 0)} \frac{\sin 2x-2x+y}{x^3+y}$$ or not. How could we check that ?? (Wondering)
  19. O

    Difficult Question in Calculus — limits and integrals

    Homework Statement (hebrew) : f(x) a continuous function. proof the following Homework Equations I guess rules of limits and integrals The Attempt at a Solution I've tried several approaches: taking ln() of both sides and using L'Hospitale Rule. Thought about using integral reduction...
  20. P

    Classical Limit of a Quantum Harmonic Oscillator

    I seem to have two approaches that I've seen and understand, but I can't quite see how they relate. 1. Write a general time evolving state as a superposition of stationary states multiplied by their exp(-iEt/h) factors, and calculate <x>. We find that <x>=Acos(wt+b) as in classical physics (in...
  21. S

    Light pipes - is there a limit

    I have seen light pipes which "conduct" sunlight from an outdoor receptor to the dark recesses of buildings ... but they all seem to be awkward bulky metallised tubes. Is it not possible to achieve the same effect with fibre optics? Could one not focus the light at the receptor into a...
  22. Mahathepp

    The limit of xye^-(x+y)^2 when x^2+y^2 approach infinity

    I try to figure it out but I can't get the answer that I need and when I look upon the solution from the book I don't understand it at all. The answer is " no limit" and there is no explanation why. The question is Determine the limit of lim (x2+y2)- -> infinity (xye-(x+y)2 in this case I use...
  23. S

    Unifying a Piecewise Function: Finding Values for Continuity

    Homework Statement Hello, thank you in advance for all help. This is a limit problem that is giving me a particularly hard time. Homework Equations For what values of a and b is f(x) continuous at every x? In other words, how to unify the three parts of a piecewise function so that there are...
  24. chikou24i

    Understanding Limit Current & Resistivity in Source Meter Machines

    We see always in source meter machines a LED which indicates the limit current. I want to know what is the limit current and what is the relationship between this later and the resistivity.
  25. B

    MHB Limit problem (sin*polynomial)

    I would really appreciate if you could help me solving this limit problem! Determine the limit without using L'Hospital's rule! $$ \lim_{x\to -2} \sin(\frac{\pi x}{2})\frac{x^2+1}{x+2} = ?$$ Thank you in advance!
  26. J

    Limit involving extinction probability of branching process

    Let x(a) be the extinction probability of a branching process whose offspring is Poisson distributed with parameter a. I need to find the limit as a approaches infinity x(a)e^a. I tried computing x(a) directly using generating functions, and I found that it's the solution to e^(a(s-1))=s, but...
  27. B

    Left-&Right-Handed Limits Explained: What Do 150 & 300 mg Mean?

    Homework Statement See attached image. Homework Equations Left- and right-handed limits. The Attempt at a Solution I know lt (t -> 12-) f = 150 mg and lt (t -> 12+) f = 300 mg, but I don't know how to explain these numbers. I'm assuming that measurements are taken and that this graph is...
  28. S

    Integration with limit of zero giving infinity - help please

    Homework Statement Integral of ∫1/x^2 (or ∫x^-2) between 1 and 0.The Attempt at a Solution I can integrate it no problem to give me -1/x or x^-1, but when I put it between the limits of 1 and 0 I get ∞-1 which is just ∞. Is this right or do I need to use L'Hopital's rule. If so, how? I'm...
  29. T

    Is the Limit as it Approaches 0 Always Infinity?

    Why is the limit not just infinity? wouldn't it be (1-infinity)/(1+infinity)?
  30. G

    Low Temp Limit: Paramagnet v. Einstein solid

    Hey everyone! So I have that the low temperature limit of a paramagnet is Ω=(Ne/Ndown)Ndown while the low temperature limit of an einstein solid is Ω=(Ne/q)q. How could I explain that these two equations are essentially the same considering their respective limits (Ndown<<N and q<<N) and that...
  31. S

    Is there a limit to wind power?

    Is there a limit to the amount of energy which can be extracted from the wind? There are a huge number of windfarms springing up around the world.. all taking energy from the wind. The assumption seems to be that this is limitless and "free". Clearly this is not possible. The question is (I...
  32. nuuskur

    Multivariable Limit: Justifying lim_{(x,y)->(0,0)} \cos{\frac{x}{\sqrt{y}}} = 1

    How do I justify that lim_{(x,y)\to (0,0)} \cos{\frac{x}{\sqrt{y}}} = 1? If I approach from the y axis, it would become lim_{y\to 0} \cos{\frac{0}{\sqrt{y}}} = 1 , but if I approach from the x axis, it would become lim_{x\to 0} \cos{\frac{x}{\sqrt{0}}} = D.N.E, no? (does not exist) Wolfram...
  33. D

    What is the Elastic Limit Measurement Unit?

    Hello guys , have a question and I can not find the answer. In what units is measured elastic limit ? Thanks A lot !
  34. T

    List of metals or alloys with fatigue limit

    I know that steel and titanium have fatigue limits. Just to clarify, metals or alloys with fatigue limits are metals that - as long as they experience pressures that lower than the limits - can last "indefinitely". Aluminum, for example, does NOT have a fatigue limit. No matter how small the...
  35. Dethrone

    MHB How can the integration limit be determined for a continuous function?

    Suppose $f$ is a continuous function on $(-\infty,\infty)$. Calculating the following in terms of $f$. $$\lim_{{x}\to{0}}f\left(\int_{0}^{\int_{0}^{x}f(y) \,dy} f(t)\,dt\right)$$
  36. lep11

    Left-handed limit of a rational function

    Homework Statement What is Lim (1+x2)/(4-x) as x approaches 4 from the left? Prove using the definition. Homework EquationsThe Attempt at a Solution Well x≠4. Function approaches positive infinity as x approaches 4 from the left side. Let m>0 and 0<x<4. Then (1+x2)/(4-x) > x2/(4-x) > x/(4-x) >...
  37. H

    Approaching 0: Rationalizing the Limit x→4 √x-4

    Homework Statement lim x→4 √x-4 I need to do something so that it is not undefined or 0. Homework EquationsThe Attempt at a Solution I tried rationalizing, but that just gave me x-4/√x+4, which would still result in an undefined answer.
  38. M

    It is said that we can not go lower than the planck limit

    Hi. My nephew asked me a good question. I am trying to understand the Planck limit. It is said that we can not go lower than the Planck limit. But if we had a an imaginary powerful microscope to see at the plank level, and if we placed 2 Planck end to end with "half" a Planck sized length...
  39. alyafey22

    MHB How to Prove a Limit in Two Variables?

    Hey MHB ! I've got a question that I am clueless how to proceed Prove that $$\Large \lim_{(x,y)\to (0,0)}(1+x^2y^2) ^{\frac{-1}{x^2+y^2}} = 1$$ Any hint would be appreciated.
  40. evinda

    MHB How can we calculate the limit?

    Hello! (Wave) I am looking at the following exercise: Let the (linear) differential equation $y'+ay=b(x)$ where $a>0, b$ continuous on $[0,+\infty)$ and $\lim_{x \to +\infty} b(x)=l \in \mathbb{R}$. Show that each solution of the differential equation goes to $\frac{l}{a}$ while $x \to...
  41. RJLiberator

    Multivariable limit problem with cos/cos

    Homework Statement Lim (x,y) --> (pi, 0) of (cos(x-y))/(cos(x+y)) Homework Equations The answer is 1 The Attempt at a Solution My answer is this: The function is continuous at the point in question, so we only need to plug in the values which result to be 1. My question here: I know this...
  42. Dethrone

    MHB What is the limit of x^n/n as n approaches infinity?

    Prove that $\lim_{{n}\to{\infty}}\frac{x^n}{n!}=0$.
  43. Dethrone

    MHB Prove $\lim_{{n}\to{\infty}}(3^n+4^n)^{1/n}=4$

    Prove that $\lim_{{n}\to{\infty}}(3^n+4^n)^{1/n}=4$
  44. Feodalherren

    Solving IVPs with Unstable Functions

    Homework Statement http://s14.postimg.org/an6f4t2ht/Untitled.png Homework EquationsThe Attempt at a Solution I'm not sure what they want me to do on the last part. I tried some googling and looking in my textbooks but I didn't find any examples. It seems to me like the function goes to...
  45. N

    Limit of e^-7x cos x: Using Squeeze Theorem for Calculus Homework

    (sorry the thread title is wrong - can a mod please change it to "Limit of e^-7x cos x?") 1. Homework Statement Find the following: \lim_{x \rightarrow \infty} e^{-7x} \cos x Homework Equations I know that [ \lim_{x \rightarrow a} f(x)g(x) ] = [ \lim_{x \rightarrow a} f(x) ] \cdot [ \lim_{x...
  46. S

    Limit as x tends to infinity, without Laurent

    Homework Statement I want to find the following limit, ## \lim_{x \rightarrow \infty } x( \sqrt{ x^{2} +9} -x) ##, without using the Laurent series Homework Equations None. The Attempt at a Solution I used the Laurent Series to expand the square root, giving ## x((x+\frac{9}{2x})-x)##, then...
  47. 22990atinesh

    How to Solve a Limit Problem Involving Logarithms?

    Suppose there is a limit ##\lim_{n \to \infty} \frac{n^{1.74}}{n \times (\log n)^9}## Taking logs both on numerator and denominator ##=\lim_{n \to \infty} \frac{1.74 \times \log n}{\log n + 9 \log \log n}## What can we say about the limit as n approaches ##\infty##
  48. Philethan

    Is the limit of functions necessarily equal to "itself"?

    As I read in the James Stewart's Calculus 7th edition, he said: My question is: Is f(x)\rightarrow 0 the same as f(x) = L? For example, f(x) = x^2 \displaystyle\lim_{x\rightarrow 5}f(x) = 25 I can say that f(x) = x^2 approaches 25 as x approaches 5. Therefore, can I say that the...
  49. L

    MHB Help With Limit: a=16, f=${x}^{.25}$

    Hello, I have this homework questions with answers. I got part (a) a=16, but part (b) f=${x}^{.25}$ I don't understand... Here is the problem: This limit represents the derivative of some function f at some number a. State this a and f $\lim_{{h}\to{0}}$$\frac{\sqrt[4]{16+h}-2}{h}$ Part a) is...
  50. K

    Optical system & diffraction limit

    can you please let me know if this sentence is true about optical systems or not? "Diffraction may limit the resolution achivable by an optical system" Thanks.
Back
Top